Hi lenihil!
To understand why answer choice (B) is incorrect and (A) correct for this one, let's take a look at both answer choices.
First, let's start with answer (A): "indicating that a particular plan would have an effect contrary to the anticipated effect." The question stem tells us that this is a method of reasoning type of question. In order to feel confident that you've chosen the correct answer on questions like this, it's important to take things that are stated generally or generically (e.g., "a particular plan") and try to give that statement substance based on the information in the passage (which specific plan is being discussed?).
We could bracket the specific information from the stimulus that corresponds to these generic words: Anika proceeds by "indicating that a particular plan [Inez's plan to get appraisers] would have an effect contrary [people would be less willing to buy because things are needlessly more costly] to the anticipated effect [the effect predicted by Inez of people being more willing to buy]." Answer choice (A) fits exactly with what Anika's response is doing--Anika claims that the plan to get appraisers would not make customers more willing to buy, because this would just drive up the price needlessly when customers are already experts enough at appraising on their own.
By contrast, answer choice (B) states that Anika proceeds by "claiming that a particular plan should not be adopted because, while effective, it would have at least one undesirable consequence." Regarding answer (B), you write:
Is that because Anika didn't say whether Inez should adopt the proposed plan or not, but instead, Anika just disagreed with Inez's prediction? Or, is that because Anika didn't think Inez's plan will be effective?
I think that both of your intuitions are correct. That is, the stimulus doesn't actually say anywhere that Anika thinks that Inez's plan will be effective. That is an example of something that should be double-checked against the stimulus. Answer choice (B) would probably be a good answer if it only eliminated the words "while effective"; since Anika doesn't explicitly acknowledge that Inez's plan would be effective, however, those two words are enough to make the answer choice incorrect because it doesn't accurately describe Anika's method of argumentation.